LSAT and Law School Admissions Forum

Get expert LSAT preparation and law school admissions advice from PowerScore Test Preparation.

User avatar
 Dave Killoran
PowerScore Staff
  • PowerScore Staff
  • Posts: 5862
  • Joined: Mar 25, 2011
|
#94757
Complete Question Explanation
(The complete setup for this game can be found here: lsat/viewtopic.php?f=270&t=8668)

The correct answer choice is (C)

The question stem asks you to identify the philosophers that, if you knew their scheduled weeks, would allow you to fully determine the schedule for the five weeks. Consider again the diagram for the game:

G2-Q10-d1.png

In this setup, K and L form one group, M and N form a second group, and O and P form a third group. As each group is fully independent of the other two groups, if you can ascertain the schedule for one member of each group, then you can determine the schedule of the other member. For example, if K is the philosopher lectured on in the first week, then L would have to be the philosopher lectured on in the second week. Thus, we need an answer that contains one of K and L, one of M and N, and one of O and P. Answer choice (C) contains one from each of the three groups, and thus answer choice (C) is correct.
You do not have the required permissions to view the files attached to this post.
 portia
  • Posts: 4
  • Joined: Jul 09, 2019
|
#66740
How do you know which philosophers determine the entire week’s schedule?
 James Finch
PowerScore Staff
  • PowerScore Staff
  • Posts: 943
  • Joined: Sep 06, 2017
|
#66758
Hi Portia,

The key to this game is understanding that there are effectively three variable sets that each contain two variables. Two of those sets can go over two slots (Slots 1+2, Slots 3+4), while the last one has only one slot to go in (Slot 5). Based on the inferences you can make from the setup plus the rules, your diagram should look something like:

Slots 1 and 2: K/L (either order)

Slots 3 and 4: M/N (either order)

Slot 5: O/P (only one of the two)

So in order to fully determine the order, we must know the placement of one of the two variables from each of the three sets. Answer choice (C) gives us that, by telling us that we'd know K, M, and O, allowing us to infer the location of the L, N and P.

Hope this clears things up!

Get the most out of your LSAT Prep Plus subscription.

Analyze and track your performance with our Testing and Analytics Package.